Un cadeau pour la rentrée — Les-mathematiques.net The most powerful custom community solution in the world

Un cadeau pour la rentrée

Pour toutes les personnes qui viennent sur le forum.

Fichier trouvé sur le web.

L'auteur Tolaso Kos précise page 4

"you are free to use the booklet as an instructive tutorial to your students"

Voir la pièce jointe
«1

Réponses

  • Bien agréable !
  • C'est moi ou les problèmes sont difficiles ?
  • Il y a de jolies intégrales et séries dans ce document. Merci beaucoup pour le partage.
  • Merci pour ce beau document qui alterne des exercices faisables et d'autres plus corsés.
  • @gaussien effectivement les exercices ne sont pas classés par ordre de difficultés.
  • Je ne comprends pas ton "2/".
    J'y vois une référence à l'autre discussion sur les exos désormais restreints aux inscrits mais même avec ça je ne saisis pas.
    C'est juste pas curiosité, pour être sûr que tu n'as pas écris un truc bizarre.


    Le message ayant changé, je raye...
  • Est-ce que pour le problème 10, la limite est 1 ??
    J"ai l'impression que numérateur et dénominateur sont équivalents à nn?
  • Attention : en factorisant par $n^n$ au numérateur et au dénominateur puis en simplifiant, on obtient un numérateur qui tend vers $1$, mais le dénominateur de tend pas vers $1$...
  • Pour le 10 il y a peut être le nombre e dans la limite.
  • Ah oui je me suis trompé au temps pour moi !
    Pour le dénominateur en factorisant par nn, j'obtiens en partant des termes de la somme de droite à gauche: 1+1/e+1/e²+...
    Donc ça ressemble fort à une série géométrique, mais je ne sais pas si on a le droit comme ça de remplacer par la somme qui est e/(e-1)...!
    Ce qui ferait comme limite : (e-1) /e...

    Vous confirmez ?
  • Une remarque :
    Rédiger que le dénominateur tend vers ce que tu dis (et je suis d'accord) n'est pas si simple je trouve.
  • @Dom : oui !! je ne suis pas satisfait de ma rédaction ! j'ai l'impression que "ça tend vers blablabla" mais ce n'est pas rigoureux du tout mon bazar ! D'ailleurs les premiers termes tendent vers 0 et je ne sais qu'en faire .
  • @totem:
    Une rédaction possible pour montrer que la limite de $\sum_{k=0}^{n-1} \left(1-\frac{k}{n}\right)^n$ est celle supputée: il suffit de montrer que pour tout $\epsilon$ il existe $K$ tel que pour $n$ assez grand $\sum_{k=K}^{n-1} \left(1-\frac{k}{n}\right)^n<\epsilon$.

    Pour arriver à ce résultat, il suffit de prendre $K$ tel que $exp(1-K)<\epsilon$ puisqu'on a alors:

    $\sum_{k=K}^{n-1} \left(1-\frac{k}{n}\right)^n=\sum_{k=K}^{n-1} \left(\frac{n-k}{n}\right)^n=\sum_{k=1}^{n-K}\left(\frac{k}{n}\right)^n=\frac{1}{n^n}\sum_{k=1}^{n-K}k^n\leq\frac{1}{n^n}\int_{1}^{n-K+1}t^ndt$ et cette intégrale tend vers $exp(1-K)$.
  • @Gaussien :

    En fait j'ai fait une comparaison série /intégrale, mais je suis bloqué pour trouver un équivalent pertinent...pourtant ça doit marcher (:D

    Tu ne dois pas faire une autre inégalité à gauche et donc un encadrement pour compléter ?
    C'est ce que j'avais commencé à faire...

    Je l'ai fait avec ta méthode, j'obtiens une intégrale "à gauche" qui tend vers exp(-K)...
  • @totem:
    Non, ce n'ai pas une comparaison série-intégrale que j'invoque. Je vais être plus précis.

    Tu veux montrer que $\lim_{n\to\infty}\sum_{k=0}^{n-1} (1-\frac{k}{n})^n=\sum_{k=0}^{\infty}e^{-k}$.

    Soit $\epsilon>0$. Soient $K$ et $N$ tels que pour $n\geq N$ on ait $\sum_{k=K}^{n-1}(1-\frac{k}{n})^n\leq\epsilon$ (un tel $K$ et un tel $N$ existent d'après mon message précédent).

    Soit également $K'\geq K$ tel que $\sum_{k=K'}^{\infty}e^{-k}\leq\epsilon$. On a alors pour $n\geq N$:
    $\lvert\sum_{k=0}^{n-1} (1-\frac{k}{n})^n-\sum_{k=0}^{\infty}e^{-k}\rvert\leq\lvert\sum_{k=0}^{K'-1} (1-\frac{k}{n})^n-e^{-k}\rvert+\lvert\sum_{k=K'}^{n-1} (1-\frac{k}{n})^n\rvert+\lvert\sum_{k=K'}^{\infty}e^{-k}\rvert\leq\lvert\sum_{k=0}^{K'-1} (1-\frac{k}{n})^n-e^{-k}\rvert+2\epsilon$.

    Puisque $K'$ est fixé, on a pour $n$ assez grand $\lvert\sum_{k=0}^{K'-1} (1-\frac{k}{n})^n-e^{-k}\rvert\leq \epsilon$ si bien que pour $n$ assez grand $\lvert\sum_{k=0}^{n-1} (1-\frac{k}{n})^n-\sum_{k=0}^{\infty}e^{-k}\rvert\leq3\epsilon$.
  • Oui ok je crois avoir compris merci. Ne faut-il pas invoquer quelque théorème du type interversion série/limite ou limite terme à terme ?
    Sans doute avec des conditions de convergence uniforme comme souvent...
  • Posons pour $n \in \N^*$, $f_n:\N \to \R^+$ d'expression $f_n(x)=(1-x/n)^n 1_{x \leq n}$ et $c$ la mesure de comptage de $\N$. La première série dont on cherche la limite est $\int_\N f_n(x) dc(x)$. Sauf erreur la suite $(f_n)_n$ tend en croissant vers $f:\N \to \R^+$ d'expression $f(x)=e^{-x}$ et un coup de Beppo-Levi ou de cv dominée et c'est fini.
  • @totem: La preuve que j'ai donnée permet d'éviter d'invoquer un quelconque théorème. Mais on peut également conclure par convergence monotone comme le suggère math2.
  • Excusez-moi je mais j'ignore ce qu'est une mesure de comptage ? Et je ne connais pas les intégrales sur N ! ça sent Lebesgue ?
  • Le 14 est original.
    $0<a<b ,\ x_1=a,\ x_2=b,$
    $x_{2n+1}=\sqrt{x_{2n}x_{2n-1}}$
    $x_{2n+2}=\dfrac{x_{2n}+x_{2n-1}}{2}$

    Trouver la limite de $x_n$
  • @ totem : oui c'est dans le cadre de la théorie de Lebesgue. La mesure de comptage sur $\N$ permet d'appliquer aux séries (à termes positifs ou absolument convergentes) les th de Lebesgue qui portent normalement sur des intégrales.
  • Tiens etanche, il me semble que c'est un classique, non ?
  • Avec $a_n=x_{2n-1}$ $b_n=x_{2n}$ c'eet la bonne vieille moyenne arithmetico geometrique de Gauss $a_{n+1}=\sqrt{a_nb_n},\ b_{n+1}=\frac{1}{2}(a_n+b_n).$ qu'on traite en montrant que $I_n=\int_0^{\pi/2}\frac{dt}{\sqrt{a_n^2\cos^2t+b_n^2\sin^2t}}$ ne depend pas de $n.$ Pour verifier cette invariance, McKean et Moll recommandent les changements de variables
    $$x=b_n\tan t,\ \ y=x+\sqrt{x^2+a_nb_n}$$ Fallait le trouver.
  • Oui, cet exo me rappelle un mauvais souvenir de colle (avec les notations $a_n$ et $b_n$ de P.), où l'examinateur m'avait laissé à l'abandon ...
  • Du coup quelle est l'expression de limite de x(n) avec a et b ?
    C'est la question demandée .
  • Bonjour,

    296. Montrer que $\displaystyle \int_0^1 \int_0^1 \int_0^1 {dxdydz \over \ln x + \ln y + \ln z} = -{1 \over 2}.$

    Voici ce que je fais.

    Existence : La fonction $\displaystyle (x,y,z) \mapsto {1 \over \ln x+\ln y+\ln z} = {1 \over \ln(xyz)}$ est définie et continue sur $\displaystyle 0<x,y,z<1.$ Lorsque deux des trois variables sont fixées dans $\displaystyle ]0,1[$, alors la fonction est équivalente à $\displaystyle x \mapsto {1 \over \ln x}, (x \to 0)$ et $\displaystyle x \mapsto {1 \over \ln y + \ln z}, (x \to 1).$ La seconde est intégrable. La première, par le changement de variables $\displaystyle x \leadsto y$ avec $\displaystyle -\ln x=y$, est équivalente à l'intégrale en $\displaystyle +\infty$ de la fonction $\displaystyle y \mapsto {e^{-y} \over y}$ qui est intégrable (par multiplication par $\displaystyle y^2$ et à la limite $\displaystyle y \to +\infty$ qui est $0$, on conclut que l'intégrande est inférieur à la fonction $\displaystyle y \mapsto {1 \over y^2}$ pour $y$ assez grand et cette dernière fonction est intégrable en $\displaystyle +\infty).$

    Changement de variables : on change les variables $\displaystyle (x,y,z) \leadsto (u,v,w)$ avec $\displaystyle -u=\ln x+\ln y+\ln z, -y=\ln y+\ln z, -w=\ln z$ qu'on inverse facilement $\displaystyle \ln x=-u+v, \ln y=-v+w, \ln z=-w$ dont le jacobien vaut $\displaystyle J=\begin{pmatrix} -e^{-u+v} & e^{-u+v} & 0 \\ 0 & -e^{-v+w} & e^{-v+w} \\ 0 &0& -e^{-w} \end{pmatrix}$ avec $\displaystyle |\det J| = e^{-u}.$
    Domaine d'intégration : De $\displaystyle w=-\ln z, 0<z<1$ on tire $\displaystyle w \in ]0, +\infty.$ De $\displaystyle w-v=\ln y, 0<y<1$ on tire $\displaystyle v \in [w, +\infty[.$ De $\displaystyle v-u=\ln x, 0<x<1$ on tire $\displaystyle u \in [v, +\infty[.$
    On a donc $\displaystyle \int_0^1 \int_0^1 \int_0^1 {dxdydz \over \ln x + \ln y + \ln z} =-\int_{0}^{+\infty} dw \int_{w}^{+\infty} dv \int_{v}^{+\infty} du {e^{-u} \over u}.$

    Etude de fonction : On note $\displaystyle \Gamma(0,z) = \int_{z}^{+\infty} du {e^{-u} \over u}, z>0$ (dont on a montré l'existence). Cette fonction est dérivable sur $\displaystyle z>0$ et $\displaystyle \Gamma'(0,z) = -{e^{-z} \over z}, z>0.$ On aura besoin des résultats suivants : $\displaystyle {d\over dz} (z \Gamma(0,z)-e^{-z})=\Gamma(0,z) + z \Gamma'(0,z)+e^{-z} = \Gamma(0,z), z>0$ et $\displaystyle {d\over dz} {1 \over 2} (z^2 \Gamma(0,z) +(1-z) e^{-z})=z \Gamma(0,z) - e^{-z}, z>0.$ Et par une intégration par partie, on obtient $\displaystyle \Gamma(0,z)= \int_{z}^{+\infty} du {e^{-u} \over u}=\ln u e^{-u}\mid_{z}^{+\infty} + \int_{z}^{+\infty} du e^{-u} \ln u = -e^{-z} \ln z + \int_{z}^{+\infty} due^{-u}\ln u , z>0.$ Comme $\displaystyle \int_{z}^{+\infty} due^{-u}\ln u \to \int_{0}^{+\infty} due^{-u}\ln u, (z \to 0)$ on a établi que $\displaystyle z^2 \Gamma(0,z) \to 0, (z \to 0).$
    Enfin, un changement de variables $\displaystyle u \leadsto t$ avec $\displaystyle t=u/z, z>0$ montre que $\displaystyle \Gamma(0,z)=\int_{1}^{+\infty} dt {-e^{-z t} \over t}$ et une intégration partie $\displaystyle \Gamma(0,z)=\int_{1}^{+\infty} dt {-e^{-z t} \over t}={e^{-z} \over z} - \int_{1}^{+\infty} dt {e^{-z t} \over zt^2}$ montre que, par simple majoration de l'intégrande par $\displaystyle t \mapsto {e^{-z} \over zt^2}$ $\displaystyle z^a\Gamma(0,z) \to 0, (z \to +\infty), a =1,2.$

    Calcul de l'intégrale : $\displaystyle -\int_{0}^{+\infty} dw \int_{w}^{+\infty} dv \int_{v}^{+\infty} du {e^{-u} \over u} =-\int_{0}^{+\infty} dw \int_{w}^{+\infty} dv \Gamma(0,v) = -\int_{0}^{+\infty} dw (v \Gamma(0,v)-e^{-v})\mid_{w}^{+\infty}=+\int_{0}^{+\infty} dw (w \Gamma(0,w)-e^{-w}) =\\ \displaystyle = {1 \over 2} e^{-w}(w^2 e^{w} \Gamma(0,w)+1-w)\mid_{0}^{+\infty}=- {1 \over 2} .$
  • Reponse a etanche: la question elementaire est de montrer que la limite existe- en montrant que ce sont des suites adjacentes. Si on ne mentionne pas l'integrale et son invariance, il serait tres difficile a un etudiant ordinaire de deviner que la limite $L$ est donnee par
    $$\frac{\pi}{2L}=\int_{0}^{\pi/2}\frac{dt}{\sqrt{a^2\cos ^2t+b^2\sin ^2t}}.$$
  • Reponse a Yves
    $$-I=\int_{[0,1]^3}\left(\int_0^{\infty}(xyz)^sds\right)dxdydz=\int_0^{\infty}\frac{ds}{(s+1)^3}ds=\frac{1}{2}.$$
  • Un laconisme efficace, monsieur P. ;-)
    Tu as mis une déesse en trop dans la dernière intégrale (:P)
  • Confucius: Quqnd tu ecris une lettre a un ami, fais une faute expres….
  • Fubini ça marche avec les intégrales multiples ? chouette...
  • Bonjour,

    @P. pour le 296. c'est bien vu (tu). Je me doutais d'un truc de ce genre, mais je n'ai pas su le voir.

    Peux-tu renouveler l'exploit pour celui-ci ?

    295. Montrer que $\displaystyle \sum_{n \geq 1} \Big( \prod_{k=1}^{n} \cos {k \pi \over n}\Big) = -{4 \over 5}.$

    Voici ce que j'ai fait.

    $\bullet$ On établit $\displaystyle \forall x \in \R, \forall m \in 2\N, \prod_{k=0}^{m-1} \cos (x+{2k\over m}\pi) = {(-1)^{m/2} - \cos mx \over 2^{m-1}}.$ On choisit $\displaystyle x=0$ et $m=2n, n \geq 1$ et on a donc $\displaystyle \prod_{k=0}^{2n-1} \cos ({k\over n}\pi) = {(-1)^{n} - 1 \over 2^{2n-1}}, n \geq 1.$
    $\bullet$ On écrit $\displaystyle \prod_{k=0}^{2n-1} = \prod_{k=0}^{n}\prod_{k=n+1}^{2n-1}$ puis, par un changement d'indice, $\displaystyle \prod_{k=n+1}^{2n-1} \cos ({k\over n}\pi) =\prod_{k=1}^{n-1} \cos (\pi+{k\over n}\pi) $ puis, comme $\displaystyle \cos ({k\over n}\pi) \mid_{k=0}=1, n \geq 1$ et $\displaystyle \cos ({k\over n}\pi) \mid_{k=n}=-1, n \geq 1$, on a $\displaystyle \prod_{k=n+1}^{2n-1} \cos ({k\over n}\pi)=(-1)^n \prod_{k=0}^{n}\cos ({k\over n}\pi), n \geq 1 .$ On a donc $\displaystyle \prod_{k=0}^{2n-1} \cos ({k\over n}\pi) = (-1)^n \big(\prod_{k=0}^{n}\cos ({k\over n}\pi) \big)^2= {(-1)^{n} - 1 \over 2^{2n-1}}, n \geq 1.$ Ce produit est donc nul pour $n$ pair puisque $\displaystyle \cos ({k\over n}\pi) \mid_{k=n/2}=0, n \in 2\N^*$ et pour $n$ impair on trouve $\displaystyle \big|\prod_{k=0}^{n}\cos ({k\over n}\pi) \big|={1 \over 2^{n-1}}, n \in 2\N+1.$ Pour trouver le signe, on coupe le produit en deux en $\displaystyle {n-1 \over 2}$ et on utilise la relation $\displaystyle \cos u = - \cos(\pi+u), u \in \R$ et alors $\displaystyle \prod_{k=0}^{n}\cos ({k\over n}\pi) =(-1)^{{n+1 \over 2}} \prod_{k=0}^{{n-1 \over 2}}\cos^2 ({k\over n}\pi).$ Finalement, on a établi que $\displaystyle \prod_{k=0}^{n}\cos ({k\over n}\pi) ={(-1)^{{n+1 \over 2}} \over 2^{n-1}}, n \in 2\N+1.$
    $\bullet$ Dans la série $\displaystyle \sum_{n \geq 1} \Big( \prod_{k=1}^{n} \cos {k \pi \over n}\Big)$ on a montré que les termes $n$ pairs ont une contribution nulle et il ne reste que les termes $n$ impairs : $n=2m+1, m \in \N$ et alors $\displaystyle \sum_{n \geq 1} \Big( \prod_{k=1}^{n} \cos {k \pi \over n}\Big) = \sum_{m \geq 0} \Big( \prod_{k=1}^{2m+1} \cos {k \pi \over 2m+1}\Big) = \sum_{m \geq 0} {(-1)^{m+1} \over 2^{2m}} = -\sum_{m \geq 0} ({-1 \over 4})^m = -{4 \over 5}.$
  • Bonjour Yves, Ca m'a l'air impeccable.
  • @P en bas de la page 5 du poly pour l'exo 14 l'auteur dit que la limite est
    $$\frac{\sqrt{b} \sqrt{b-a}}{2\ln(\sqrt{b}-\sqrt{b-a})-\ln(a)}$$
    Comment le prouver P ? Merci
  • 19 7 on note <n> l'entier le plus proche de $\sqrt{n}$ calculer la somme de la serie
    $$\sum_{n=1}^{\infty}\frac{2^{<n>}+2^{-<n>}}{2^n}$$
  • Comment attaque-t-on les problèmes comme le 7 avec des racines de 6 imbriquées ?
  • Pour le 7 $f(x)=\sqrt{x+6}$
  • En effet, c'est formellement une suite : $u_{n+1}=f(u_n)$.
  • OK donc il faut donc montrer que u est monotone,bornée donc u converge vers un des points fixes de f ?
  • Il suffit de cela oui... ;-)
  • En effet c'est un très beau cadeau.
    Certains énoncés sont très classiques comme justement ces racines de 6 imbriquées (problème 7, page 5), et d'autres extrêmement déroutants.
    Merci à Etanche.
    Bonne journée.
    Fr. Ch.
  • OK donc ça converge vers 3 (et vite en plus !)

    Et le problème 5 des idées ? il est ardu celui là ::o

    J'ai très envie d'utiliser la formule de Stirling, mais avec le sinus qui n'est pas de signe constant je n'ose pas.
  • Le problème 5 est en fait « un grand classique », au sens où je le connais depuis ma taupe et qu'il est certainement dans les recueils d'oraux sous différents avatars (ex. : nature de la série $\sum\sin(\pi \mathrm{e}n!)$).

    La clé : $\displaystyle\mathrm{e}=\sum_{k\ge1}\frac{1}{k!}$ ; coupe la somme vers le terme $k=n$.
  • Merci pour ce cadeau.

    Pour le 197: remarquer que $k=<n>$ est équivalent à $k^2-k+1\leq n\leq k^2+k$.
    Cela conduit à une somme télescopique.

    On peut généraliser en remplaçant $2$ par $x>1$.
  • @Math Coss :

    J'ai essayé comme tu m'as dit, j'ai coupé la somme avec $e= Sn + Rn$, puis formule $\sin(a+b) = \sin(a) \cos(b) + \sin(b) \cos(a)$ , mais après je suis bloqué de chez bloqué... Comment gérer un sinus d'une somme ?

    Bon alors $ \sin(2\pi en!) = \sin(2\pi n!Rn)\cos(2\pi n!Sn) + \sin(2\pi n!Sn)\cos(2\pi n!Rn)$

    $n!Rn$ ça tend vers 0 et c'est équivalent à $1/n$

    Donc $\sin(2\pi n!Rn)\sim 2\pi/n$ et $\cos(2\pi n!Rn)\sim 1$ ; par contre les termes avec $Sn$ je ne sais qu'en faire...
  • C'est l'idée principale. Note que je n'ai pas dit en combien il faut couper... Je trouve plus commode de faire trois paquets. En notant \[S_n=\sum_{k=0}^n\frac{1}{k!}\quad\text{et}\quad R'_n=\sum_{\ell=n+2}^{+\infty}\frac1{\ell!},\] on a \[n!\mathrm{e}=n!S_n+\frac{1}{n+1}+n!R'_n.\]D'un côté, $n!S_n\in\N$ donc on peut effacer $2\pi n!S_n$ dans l'argument du sinus, et de l'autre, $\lim_{n\to\infty}(n+1)!R'_n=0$ (Taylor-Lagrange par exemple, ou directement). En gros, la suite doit être équivalente à $\sin\frac{2\pi}{n+1}$.

    Pour la version « $\sum \sin(\pi\mathrm{e}n!)$ », le terme général est de la forme $(-1)^n/n+O(1/n^2)$ (à une constante multiplicative près, $\pi$ ou $\pi/2$ ?), donc la série est semi-convergente.

    NB : Merci de faire l'effort d'encadrer tes formules entre dollars. L'étape suivante, c'est de mettre des backslashes \ : devant les fonctions usuelles, ça les met en romain (\sin donne $\sin$, \ln donne $\ln$, etc.), devant pi (\pi donne $\pi$), devant sum (\sum donne $\sum$). Et puis il y a les exposants, marqués par ^, et les indices, marqués par _ (ainsi, R_n^2 donne $R_n^2$).
  • Merci !
    Pour la série que tu proposes (sans le 2) la constante doit être Pi, mais je suis toujours embêté pour manipuler des équivalents avec des séries alternées...le O(1/n²)a le dernier mot c'est ça ??
  • Tiens, une variante qui me revient en tête : nature de la série de terme général $\sin\left(\pi\sqrt{n^2+1}\right)$ ?
  • Plus facile celle là ! Semi-convergente aussi .
  • Bonjour,

    287. Calculer $\displaystyle I = \int_{0}^{+\infty} \ln^2 ({x \over x^2+1}) {1 \over (x^2+1)^2} dx.$

    Voici ce que je fais.

    Existence : La fonction $\displaystyle x \mapsto \ln^2 ({x \over x^2+1}) {1 \over (x^2+1)^2}$ est définie et continue sur $\displaystyle ]0, +\infty[$ et l'intégrale est impropre aux deux bornes. Comme $\displaystyle \ln^2 ({x \over x^2+1}) {1 \over (x^2+1)^2} \sim \ln^2 x, (x \to 0)$ qui est intégrable en $0$, et $\displaystyle x^2 \times \ln^2 ({x \over x^2+1}) {1 \over (x^2+1)^2} \sim {\ln^2 x \over x^2} \to 0, (x \to +\infty)$ ce qui permet de majorer l'intégrande par la fonction $\displaystyle x \mapsto {1 \over x^2}$ qui est intégrable en $+\infty.$

    Changements de variables :
    $\bullet$ On change les variables $\displaystyle x \leadsto y$ avec $\displaystyle x={1 \over y}, y>0$ : $\displaystyle I = \int_{0}^{+\infty} \ln^2 ({y \over y^2+1}) {y^2 \over (y^2+1)^2} dy.$ Et on a diminué le degré du dénominateur par $\displaystyle 2I = \int_{0}^{+\infty} \ln^2 ({x \over x^2+1}) {1 \over x^2+1} dx.$
    $\bullet$ On change les variables $\displaystyle x \leadsto z$ avec $\displaystyle x = \tan z, 0<z<{\pi \over 2}$ : $\displaystyle 2I = \int_{0}^{{\pi \over 2}} dz \ln^2 (\sin z \cos z) dz.$
    $\bullet$ On change les variables $\displaystyle z \leadsto t$ avec $\displaystyle 2z=t$ : $\displaystyle 4I =\int_{0}^{\pi} \ln^2 {\sin t \over 2} dt = \int_{0}^{\pi} \ln^2 \sin t dt - 2 \ln 2 \int_{0}^{\pi} \ln \sin t dt + \pi \ln^2 2.$

    Calculs d'intégrales : Le calcul de $\displaystyle \int_{0}^{\pi} \ln^2 \sin t dt$ et de $\displaystyle \int_{0}^{\pi} \ln \sin t dt$ a été fait dans ce fil : http://www.les-mathematiques.net/phorum/read.php?4,1674916,1678588#msg-1678588. Ici, la borne supérieure permet un calcul facile.
    $\bullet$ De la formule d'Euler $\displaystyle \sin z = {e^{iz} - e^{-iz}\over 2i}, z\in \C$, de la formule de Moivre $\displaystyle (e^{iz})^k = \cos kz+i \sin kz, z\in \C, k \in \N$, de la série $\displaystyle \ln(1-z) = -\sum_{k \geq 1} {z^k \over k}, z \in \C, |z|<1$ et du logarithme principal on tire $\displaystyle \ln \sin x = \ln ({i e^{-ix} \over 2}(1-e^{2ix}))= -\ln 2 - \sum_{k \geq 1} {\cos 2kx \over k} + i \big({\pi \over 2}-x - \sum_{k \geq 1} {\sin 2kx \over k} \big), x \in \R.$
    $\bullet$ On ne retient que la partie réelle pour calculer $\displaystyle \int_{0}^{\pi} \ln \sin t dt = -\pi \ln 2$ puisque $\displaystyle \int_{0}^{\pi} \cos( 2kt) dt = 0, k \in N^*.$
    $\bullet$ On élève au carré la relation trouvée pour calculer $\displaystyle \int_{0}^{\pi} \ln^2 \sin t dt =\int_{0}^{\pi} \big( \ln^2 2 + 2 \ln 2 \sum_{k \geq 1} {\cos 2kt \over k} + \sum_{k \geq 1} \sum_{k' \geq 1} {\cos 2kt \over k}{\cos 2k't \over k'}\big) dt.$ On distingue le cas $k=k'$ des autres et par la formule de duplication $\displaystyle \cos 2z = 2 \cos^2z-1, z\in \C$ et la relation $\displaystyle 2\cos (2kt)\cos (2k't)=\cos(2(k+k')t) + \cos(2(k-k')t), (k,k')\in \N^*\times \N^*$ on calcule $\displaystyle \int_{0}^{\pi} \cos^2( 2kt) dt = {\pi \over 2}, k \in N^*$ et $\displaystyle \int_{0}^{\pi} \cos (2kt)\cos (2k't) dt = 0, k \neq k', (k,k')\in \N^*\times \N^*.$ On a donc établi que $\displaystyle \int_{0}^{\pi} \ln^2 \sin t dt = \pi \ln^2 2 + {\pi \over 2} \zeta(2)$ avec $\displaystyle \zeta(2) = \sum_{k \geq 1} {1 \over k^2}={\pi^2 \over 6}.$
    Le résultat : On reporte dans la relation précédente pour obtenir $\displaystyle I = \int_{0}^{+\infty} \ln^2 ({x \over x^2+1}) {1 \over (x^2+1)^2} dx=\pi \ln^2 2 + {\pi \over 8} \zeta(2) = \pi \ln^2 2 + {\pi^3 \over 48}.$
Connectez-vous ou Inscrivez-vous pour répondre.
Success message!